How Do You Prove a Function is Not Uniformly Continuous?

Click For Summary
To prove that a function f is not uniformly continuous on a set X, it is necessary to demonstrate the existence of an ε > 0 and sequences (p_n) and (q_n) in X such that the distance d_X(p_n, q_n) approaches 0 while the distance d_Y(f(p_n), f(q_n)) remains at least ε. The proof begins by showing that if d(p_n, q_n) approaches 0, then f cannot be uniformly continuous, as it contradicts the definition of uniform continuity. Conversely, if f is not uniformly continuous, it implies that for every δ > 0, there exist points x and y in X such that d_X(x, y) < δ leads to d_Y(f(x), f(y)) ≥ ε. A concrete example, such as f(x) = x^2, illustrates this concept, where specific sequences can be constructed to satisfy the required conditions. The discussion emphasizes the importance of defining appropriate sequences to complete the proof effectively.
Incand
Messages
334
Reaction score
47

Homework Statement


Let ##f:X \to Y##. Show that
##f## not uniform continuous on ##X## ##\Longleftrightarrow## ##\exists \epsilon > 0## and sequences ##(p_n), (q_n)## in ##X## so that ##d_X(p_n,q_n)\to 0 ## while ##d_Y(f(p_n),f(q_n))\ge \epsilon##.

Homework Equations


Let ##f:X\to Y##. We say ##f## is uniform continuous on ##X## if ##\forall \epsilon >0 \exists \delta > 0## so that
##d_Y(f(x),f(y))< \epsilon## ##\forall x,y\in X## for which ##d_X(x,y)< \delta##.

The Attempt at a Solution


I was hoping someone could take a look at my proof and check if it's correct or not. I found the second part especially hard to formulate so I'm mostly unsure about that part.

Starting with ##\Longleftarrow##
##d(p_n,q_n) \to 0## means by definition that
##\forall \delta > 0## ##\exists N## so that ##d(p_n,q_n) < \delta## for ##n \ge N##. But then ##f## can't be uniform continuous since the points ##p_n, q_n \in X## and ##d_Y(f(p_n),f(q_n))\ge \epsilon_0## we have a counter example to
##d(x,y)<\delta \Longrightarrow d(f(x),f(y)< \epsilon## if we take a ##\epsilon < \epsilon_0##.

##\Longrightarrow##
That ##f## is not uniform continuous implies that ##\exists \epsilon >0## so that ##\forall \delta > 0## there exists ##x,y\in X## so that
##d_X(x,y)< \delta \Longrightarrow d_Y(f(x),f(y))\ge \epsilon##. Let's select some ##x,y## satisfying this and set ##p_n = x## and ##q_n =y## and we're done.
 
Physics news on Phys.org
Incand said:
##\Longrightarrow##
That ##f## is not uniform continuous implies that ##\exists \epsilon >0## so that ##\forall \delta > 0## there exists ##x,y\in X## so that
##d_X(x,y)< \delta \Longrightarrow d_Y(f(x),f(y))\ge \epsilon##. Let's select some ##x,y## satisfying this and set ##p_n = x## and ##q_n =y## and we're done.

You're not finished. You haven't defined the sequences p_n and q_n.

Maybe it helps to look at a concrete case: f(x) = x^2. This is not uniformly continuous. Pick \epsilon = 1. Then for any \delta &gt; 0, we can find an x and y such that |x-y| &lt; \delta, but |f(x) - f(y)| &gt; \epsilon. You just let x = \frac{1}{\delta}, and let y = x+\frac{\delta}{2}. Then even though |x-y| &lt; \delta, |x^2 - y^2| = 1 + \frac{\delta^2}{4} &gt; \epsilon.

So it's not uniformly continuous. But how does your argument show that there is a sequence p_n and a sequence q_n such that |p_n - q_n| \rightarrow 0 but |(p_n)^2 - (q_n)^2| &gt; \epsilon?

(It's not hard to come up with such a sequence, but it doesn't seem to follow immediately from your proof.)
 
  • Like
Likes Incand
Good catch there! I tried to fix the problem:
So we have ##d_X(x,y)<\delta \Longrightarrow d_Y(f(x),f(y))\ge \epsilon##.
Set ##N= floor(1+1/\delta)## Then ##d_X(x_n,y_n)\le 1/n < \delta## for ##n\ge N## implies ##d_Y(f(x),f(y))\ge \epsilon##.
So for each ##n## select ##x_n,y_n## satisfying the above which will then form two sequences ##x_n## and ##p_n## for which ##d_X(x_n,y_n)< 1/n \to 0##.

For your example we could set ##p_n = n+1/n## and ##q_n = n## then ##|p_n-q_n| = 1/n \to 0## while ##|(n+1/n)^2-n^2| = 2+1/n^2 \to 2##.
 
In the proof above we can shift the indexes to start counting from ##1,2,\dots## by setting ##p_1 = x_N, p_2 = x_{N+1}, \dots ##
 
Question: A clock's minute hand has length 4 and its hour hand has length 3. What is the distance between the tips at the moment when it is increasing most rapidly?(Putnam Exam Question) Answer: Making assumption that both the hands moves at constant angular velocities, the answer is ## \sqrt{7} .## But don't you think this assumption is somewhat doubtful and wrong?

Similar threads

  • · Replies 40 ·
2
Replies
40
Views
4K
  • · Replies 14 ·
Replies
14
Views
1K
  • · Replies 2 ·
Replies
2
Views
1K
  • · Replies 7 ·
Replies
7
Views
1K
Replies
23
Views
3K
  • · Replies 5 ·
Replies
5
Views
2K
  • · Replies 8 ·
Replies
8
Views
3K
  • · Replies 6 ·
Replies
6
Views
2K
  • · Replies 4 ·
Replies
4
Views
2K
  • · Replies 8 ·
Replies
8
Views
4K